Difference between revisions of "2018 AMC 10B Problems/Problem 13"
m (fixed an issue with solution 2 where it showed (blank) mod 101.) |
(→Solution 6 (Complex Numbers)) |
||
(11 intermediate revisions by 2 users not shown) | |||
Line 18: | Line 18: | ||
<br> | <br> | ||
− | ==Solution 2== | + | ==Solution 2 (Similar to solution 1)== |
Note that <math>10^{2k}+1</math> for some odd <math>k</math> will satisfy <math>10^{2k}+1 \equiv -1\pmod {101}</math>. Each <math>2k \in \{2,6,10,\dots,2018\}</math>, so the answer is <math>\boxed{\textbf{(C) } 505}</math> | Note that <math>10^{2k}+1</math> for some odd <math>k</math> will satisfy <math>10^{2k}+1 \equiv -1\pmod {101}</math>. Each <math>2k \in \{2,6,10,\dots,2018\}</math>, so the answer is <math>\boxed{\textbf{(C) } 505}</math> | ||
Line 28: | Line 28: | ||
==Solution 5== | ==Solution 5== | ||
Note that <math>101=x^2+1</math> and <math>100...0001=x^n+1</math>, where <math>x=10</math>. We have that <math>\frac{x^n+1}{x^2+1}</math> must have a remainder of <math>0</math>. By the remainder theorem, the roots of <math>x^2+1</math> must also be roots of <math>x^n+1</math>. Plugging in <math>i,-i</math> to <math>x^n+1</math> yields that <math>n\equiv2\mod{4}</math>. Because the sequence starts with <math>10^2+1</math>, the answer is <math>\lceil 2018/4 \rceil=\boxed{\textbf{(C) } 505}</math> | Note that <math>101=x^2+1</math> and <math>100...0001=x^n+1</math>, where <math>x=10</math>. We have that <math>\frac{x^n+1}{x^2+1}</math> must have a remainder of <math>0</math>. By the remainder theorem, the roots of <math>x^2+1</math> must also be roots of <math>x^n+1</math>. Plugging in <math>i,-i</math> to <math>x^n+1</math> yields that <math>n\equiv2\mod{4}</math>. Because the sequence starts with <math>10^2+1</math>, the answer is <math>\lceil 2018/4 \rceil=\boxed{\textbf{(C) } 505}</math> | ||
+ | |||
+ | |||
+ | ==Solution 6 (Complex Numbers)== | ||
+ | |||
+ | The <math>n</math>th term of the sequence is <math>10^n+1</math> for <math>n\ge 2</math> (since <math>10^2+1=101</math>, <math>10^3+1=1001</math>, etc.). We seek those <math>n</math> for which <cmath>101 \mid (10^n+1).</cmath>Let <math>x = 10</math>. Note that <math>101=10^2+1</math>, so this is equivalent to the polynomial divisibility | ||
+ | |||
+ | <cmath>x^2+1 \mid x^n+1</cmath> | ||
+ | |||
+ | Over <math>\mathbb{C}</math>, <math>x^2+1</math> has roots <math>i,-i</math>. Thus <math>x^2+1 \mid x^n+1</math> if both <math>i,-i</math> are roots of <math>x^n+1</math>, since, | ||
+ | <math>i^n=-1 \quad \text{and} \quad (-i)^n=-1.</math> | ||
+ | Because <math>i^4=1</math>, we have <math>i^n=-1</math> exactly when <math>n\equiv 2 \pmod{4}</math>. For such <math>n</math>, <math>(-i)^n=(-1)^n i^n=i^n=-1</math> as well, so the condition holds precisely when | ||
+ | <math>n \equiv 2 \pmod{4}.</math> | ||
+ | |||
+ | Among the first <math>2018</math> terms of the sequence (which correspond to <math>n=2,3,\dots,2019</math>), the values of <math>n</math> congruent to <math>2 \pmod{4}</math> are | ||
+ | <math>2,6,10,\dots,2018</math>. | ||
+ | This is an arithmetic progression with common difference <math>4</math>; the count is | ||
+ | |||
+ | <math>\frac{2018-2}{4}+1=\frac{2016}{4}+1=504+1=505</math>. | ||
+ | Therefore, exactly <math>\boxed{505}</math> of the first <math>2018</math> numbers are divisible by <math>101</math>, which corresponds to choice <math>\boxed{\textbf{(C)}}</math>. | ||
+ | |||
+ | ~ Aeioujyot | ||
==See Also== | ==See Also== |
Latest revision as of 15:20, 27 September 2025
Contents
Problem
How many of the first numbers in the sequence
are divisible by
?
Solution 1
The number is divisible by 101 if and only if
. We note that
, so the powers of 10 are 4-periodic mod 101.
It follows that if and only if
.
In the given list, , the desired exponents are
, and there are
numbers in that list.
Solution 2 (Similar to solution 1)
Note that for some odd
will satisfy
. Each
, so the answer is
Solution 3
If we divide each number by , we see a pattern occuring in every 4 numbers.
. We divide
by
to get
with
left over. Looking at our pattern of four numbers from above, the first number is divisible by
. This means that the first of the
left over will be divisible by
, so our answer is
.
Solution 4
Note that is divisible by
, and thus
is too. We know that
is divisible and
isn't so let us start from
. We subtract
to get 2. Likewise from
we subtract, but we instead subtract
times
or
to get
. We do it again and multiply the 9's by
to get
. Following the same knowledge, we can use mod
to finish the problem. The sequence will just be subtracting 1, multiplying by 10, then adding 1. Thus the sequence is
. Thus it repeats every four. Consider the sequence after the 1st term and we have 2017 numbers. Divide
by four to get
remainder
. Thus the answer is
plus the 1st term or
.
Solution 5
Note that and
, where
. We have that
must have a remainder of
. By the remainder theorem, the roots of
must also be roots of
. Plugging in
to
yields that
. Because the sequence starts with
, the answer is
Solution 6 (Complex Numbers)
The th term of the sequence is
for
(since
,
, etc.). We seek those
for which
Let
. Note that
, so this is equivalent to the polynomial divisibility
Over ,
has roots
. Thus
if both
are roots of
, since,
Because
, we have
exactly when
. For such
,
as well, so the condition holds precisely when
Among the first terms of the sequence (which correspond to
), the values of
congruent to
are
.
This is an arithmetic progression with common difference
; the count is
.
Therefore, exactly
of the first
numbers are divisible by
, which corresponds to choice
.
~ Aeioujyot
See Also
2018 AMC 10B (Problems • Answer Key • Resources) | ||
Preceded by Problem 12 |
Followed by Problem 14 | |
1 • 2 • 3 • 4 • 5 • 6 • 7 • 8 • 9 • 10 • 11 • 12 • 13 • 14 • 15 • 16 • 17 • 18 • 19 • 20 • 21 • 22 • 23 • 24 • 25 | ||
All AMC 10 Problems and Solutions |
These problems are copyrighted © by the Mathematical Association of America, as part of the American Mathematics Competitions.